LSAT and Law School Admissions Forum

Get expert LSAT preparation and law school admissions advice from PowerScore Test Preparation.

 Administrator
PowerScore Staff
  • PowerScore Staff
  • Posts: 8916
  • Joined: Feb 02, 2011
|
#41656
Complete Question Explanation
(The complete setup for this game can be found here: lsat/viewtopic.php?t=8572)

The correct answer choice is (C)

As the last question in this section, the Rule Substitution question presented a considerable challenge for many test-takers who did not have the time to answer it. The actual question, however, was not nearly as difficult as it may appear at first.

The stem asks us to replace the first rule of the game, which required G to start on the hour rather than the half hour. Our job is to substitute that rule with a logically equivalent condition, which would have the exact same effect on the order in which the houses are shown. While the wording of the correct answer can be difficult-to-impossible to predict, the four incorrect answer choices will either present rules that were not part of the original rule set (also known as Additional Effects conditions), or else they will only partially constrain the variables in the rule being substituted (i.e. Partial Match conditions).

Since Additional Effects are easier to spot, look for answer choices that contain such conditions first. Our task is made significantly easier by the use of Templates: if an answer choice contains a statement that is violated in at least one template, then this answer choice will contain an Additional Effect rule that restricts the game too much.

Notice, for instance, that answer choices (B) adds a condition extraneous to the original rule set, because the rule being substituted does not require G to be either the first or the fifth program: if G starts at 2:00, which is allowed by the original rule, then G would be the third program (Templates 2A, 2B).

Likewise, answer choice (D) states that if G is shown third, then R is shown first. Template 2A disproves that statement, showing that either R or W can shown first. Therefore, answer choice (D) also contains an Additional Effect rule.

Finally, answer choice (E) suggests that if G is not shown first, then it is shown later than T. This clearly does not need to be true, as shown in Template 2B, where G is shown third, and T—fifth. So, answer choice (E) contains a condition extraneous to the original rule set.

We are left with answer choices (A) and (C).

According to answer choice (A), G is not shown immediately before T. The templates bear this out, as a GT Block cannot be formed in any of them. However, this rule does not prevent G from starting on the half hour. For instance, the following solution would be possible:
PT65_D11 LG Explanations_game_#4_#23_diagram 1.png
Since such a solution is disallowed by the original rule being substituted, answer choice (A) contains a Partial Match condition that only partially constrains G.

Answer choice (C) is the correct answer choice, as it is the only remaining contender. The rule disallows G from being the second or the fourth program shown, which is consistent with the six templates: G is never the second or the fourth program in any of them. Answer choice (C) does not therefore contain an Additional Effects rule. Does answer choice (C) contain a Partial Match?

If G cannot be the second or the fourth program, it must be the first, the third, or the fifth program. This is precisely what we determined in our original three templates:
PT65_D11 LG Explanations_game_#4_#23_diagram 2.png
Thus, the effect of the new condition is identical to that of the original rule, and answer choice (C) does not contain a Partial Match. It is therefore the correct answer choice.
You do not have the required permissions to view the files attached to this post.

Get the most out of your LSAT Prep Plus subscription.

Analyze and track your performance with our Testing and Analytics Package.